Galois Theory - irreducibility over Q

  • Thread starter Thread starter Kate2010
  • Start date Start date
  • Tags Tags
    Theory
Click For Summary
To show that xn - a is irreducible over Q for a > 1 as a product of distinct primes and for all n ≥ 2, the rational root theorem can be considered, although it only confirms the absence of linear factors. Eisenstein's criterion is also applicable and can be a useful tool for establishing irreducibility in this case. The initial confusion stemmed from the belief that these methods might not be relevant, but further discussion clarified their usefulness. Overall, the problem can be approached effectively using these established criteria. Understanding these tools simplifies the process of proving irreducibility in polynomial equations.
Kate2010
Messages
134
Reaction score
0

Homework Statement



If a>1 is a product of distinct primes, show that xn-a is irreducible over Q for all n ≥ 2.

Homework Equations





The Attempt at a Solution



I am not really sure how to start this problem. Can anyone point me in the right direction?

I know tests for irreducibility for example Eisensteins Criterion or reduction modulo p but I don't think that these are helpful here?

Thanks for any help.
 
Physics news on Phys.org
Hi Kate2010! :smile:

How about the rational root theorem?
 
I like Serena said:
Hi Kate2010! :smile:

How about the rational root theorem?

That would only tell you it doesn't have any linear factors. I'm a little confused why Kate2010 thinks Eisenstein's criterion isn't applicable.
 
Dick said:
That would only tell you it doesn't have any linear factors. I'm a little confused why Kate2010 thinks Eisenstein's criterion isn't applicable.

Right.
Just looked up Eisenstein's criterion.
Looks like a good one. :)
 
Thanks guys - I was trying to make things more complicated than they were.
 
Question: A clock's minute hand has length 4 and its hour hand has length 3. What is the distance between the tips at the moment when it is increasing most rapidly?(Putnam Exam Question) Answer: Making assumption that both the hands moves at constant angular velocities, the answer is ## \sqrt{7} .## But don't you think this assumption is somewhat doubtful and wrong?

Similar threads

  • · Replies 1 ·
Replies
1
Views
2K
  • · Replies 8 ·
Replies
8
Views
2K
  • · Replies 1 ·
Replies
1
Views
1K
Replies
48
Views
4K
  • · Replies 6 ·
Replies
6
Views
2K
  • · Replies 3 ·
Replies
3
Views
2K
  • · Replies 28 ·
Replies
28
Views
5K
  • · Replies 1 ·
Replies
1
Views
2K
  • · Replies 11 ·
Replies
11
Views
2K
  • · Replies 3 ·
Replies
3
Views
2K